The Hardest Integral From The Hardest Test (Putnam Exam)

แชร์
ฝัง
  • เผยแพร่เมื่อ 18 ต.ค. 2024
  • 🎥 The Putnam's Toughest Integral Solved! 📚🔥
    Join me as I tackle one of the most challenging integrals from the Putnam Exam: the infamous integral x^-x ! In this video, we'll break down the problem step-by-step. Whether you're a math enthusiast, a student preparing for the Putnam Exam, or just curious about advanced calculus, this video is for you!
    🔍 What You'll Learn:
    The Putnam Exam and its significance
    Different approaches to solving integrals
    Detailed solution to the integral x^-x
    Tips and tricks for solving complex math problems.
    👍 Don't forget to like, comment, and subscribe for more math content!
    🔔 Hit the bell icon to get notified whenever I upload new videos.
    📢 Share this video with your friends who love math and challenges!
    📱 Follow me on social media:
    Instagram: @jagosprivatelife
    📧 Contact me for collaboration or inquiries
    🔍 Relevant Hashtags:
    #PutnamExam #IntegralCalculus #MathChallenge #AdvancedMath #Calculus #Mathematics #PutnamCompetition #MathTutorial #LearnMath #STEM #Education #MathVideo
    Thank you for watching, and happy solving!

ความคิดเห็น • 211

  • @Jagoalexander
    @Jagoalexander  2 หลายเดือนก่อน +37

    Thank you for 1k subscribers! Onwards and upwards.

  • @jujdj6214
    @jujdj6214 2 หลายเดือนก่อน +57

    i think thats the best video for an introduction to your channel i could get

    • @Jagoalexander
      @Jagoalexander  2 หลายเดือนก่อน +10

      I'm glad you like it :)

    • @improbabilty
      @improbabilty 2 หลายเดือนก่อน

      I want to say the sammmee❤

    • @Jagoalexander
      @Jagoalexander  2 หลายเดือนก่อน +1

      @@improbabilty

  • @PrometheusZandski
    @PrometheusZandski 2 หลายเดือนก่อน +8

    You are a gentleman and a scholar, sir. Apology accepted. I personally don't want music to distract my thought process when doing math.
    Absolutely amazing solution.

    • @Jagoalexander
      @Jagoalexander  2 หลายเดือนก่อน +2

      I appreciate that

  • @OliverGoodman-todd
    @OliverGoodman-todd 2 หลายเดือนก่อน +193

    that guy definitely had anger issues haha

    • @knowledgestar1081
      @knowledgestar1081 2 หลายเดือนก่อน +4

      People don't have ability to make videos like this TH-camr so they think that using such great words makes them powerfull.😂 Actually people who lack vocabulary start using such words.

    • @knowledgestar1081
      @knowledgestar1081 2 หลายเดือนก่อน +3

      Whatever you do people have some reason to scold you

    • @MrGLA-zs8xt
      @MrGLA-zs8xt หลายเดือนก่อน

      Yes you better not to make him angry

  • @naturallyinterested7569
    @naturallyinterested7569 2 หลายเดือนก่อน +92

    The final value is roughly 1.3

    • @zachariastsampasidis8880
      @zachariastsampasidis8880 2 หลายเดือนก่อน +8

      Should be transcendental due to Liouville's theorem

    • @Yuriko_Kuroki
      @Yuriko_Kuroki 2 หลายเดือนก่อน +8

      1.29129 actually which is also pretty elegant

    • @Mathslover666
      @Mathslover666 2 หลายเดือนก่อน +6

      It's 1.29128599706....approx I graph it on desmos. Lol

    • @Mathslover666
      @Mathslover666 2 หลายเดือนก่อน +2

      ​@@zachariastsampasidis8880 common sense bro 😗

    • @Kainnable
      @Kainnable หลายเดือนก่อน

      NIntegrate[x^-x, {x,0,1}] = 1.29129

  • @bontear2628
    @bontear2628 2 หลายเดือนก่อน +53

    that comment part felt more like a chess review

  • @johnlv12
    @johnlv12 2 หลายเดือนก่อน +17

    There is exactly one "k" in the video- at 12:58. I was able to follow everything easily because you said everything as you were writing. I just point it out because I thought it was funny that one time you realized the "k" looks like a "u" and made a special effort to make it look good, but immediately reverted back after that. Everything is good.
    On a separate issue, I don't doubt that this is the solution that would get 10/10 points on the exam, but to me it doesn't look any simpler than the original integral. They are both infinite sums that have to be evaluated numerically. The solution converges quickly and even the first handful of terms are good enough to approximate the entire sum, so that's good if you were trying to get a numerical approximation. My problem is with the statement of the problem- it could have said show that the integral equals the sum.

    • @Hand_of_the_king
      @Hand_of_the_king 2 หลายเดือนก่อน

      There is one at 18:36 too

    • @Jagoalexander
      @Jagoalexander  2 หลายเดือนก่อน +3

      😭😭😭😭 I think this is because it's just been exam season so I got used to just writing really fast and not caring about handwriting

  • @ShenghuiYang
    @ShenghuiYang 2 หลายเดือนก่อน +94

    Sophomore's dream.

    • @sydhenderson6753
      @sydhenderson6753 2 หลายเดือนก่อน +3

      And true to life, I proved it (and the corresponding integral with x^x) in my sophomore year.

  • @awkwardhamster8541
    @awkwardhamster8541 2 หลายเดือนก่อน +23

    whys ur comment section filled with weird ppl ? ur videos r very well explained and ur voice is very calming :)

    • @DeadJDona
      @DeadJDona 2 หลายเดือนก่อน

      @@awkwardhamster8541 dats y we here

  • @dalisabe62
    @dalisabe62 หลายเดือนก่อน +1

    When I first saw the problem, I was tempted to use the definition of the integral as the limit of the sum as the index goes to infinity. It’s nice to see at least that the concept of integration is the limit of an infinite sum of a discrete sum.

  • @SlightSmile
    @SlightSmile 2 หลายเดือนก่อน +21

    I love the "complaining and explanation" that people seem to complain about. Just keep doing your thing lol.
    Very nice solution btw.

    • @Jagoalexander
      @Jagoalexander  2 หลายเดือนก่อน

      Thanks mate !

    • @colt4667
      @colt4667 หลายเดือนก่อน

      I wish his penmanship were a little better.

  • @txikitofandango
    @txikitofandango 2 หลายเดือนก่อน +87

    Nice video but i thought it could use more music to jazz it up

    • @harshnambiar
      @harshnambiar 2 หลายเดือนก่อน

      Maybe a lot of people would have downvoted if the music was annoying

    • @gdcuaer4076
      @gdcuaer4076 2 หลายเดือนก่อน

      Yea we need to jazz this vid up!!!!!!
      Would really help

  • @jeffberkeley
    @jeffberkeley 28 วันที่ผ่านมา

    Yes, thank you, a most surprising result.

  • @cyrilanderson
    @cyrilanderson 2 หลายเดือนก่อน +2

    Nice. I think Michael Penn did one on this integral too.
    It looks like it would converge extremely quickly too, which would make it easy to approximate to a chosen level of precision.

  • @quite_unknown_1
    @quite_unknown_1 2 หลายเดือนก่อน +2

    I came for the math, but I enjoyed the analysis of dashxdr’s comment the most

  • @dalisabe62
    @dalisabe62 หลายเดือนก่อน +2

    Seeing limited integrals as the infinite sum of a converging discrete sequence helps conceptualize the process incorporated in this solution. After all, the only difference between the final left and right hand in the answer is the shift of the integration symbol to the summation symbol and the change of the limits of integration and summation (from zero to infinity) to (one to infinity) respectively. That makes perfect sense because X to the power of negative X has undetermined value when X = 0 or = 1 by definition. Of course, the original problem was written in terms of X not alpha, so the back substitution needs to be made.

    • @chibuzorbethel3292
      @chibuzorbethel3292 หลายเดือนก่อน

      Exactly what I'm thinking; back substitution to return the result in terms of the original variable x

  • @LouisEmery
    @LouisEmery 2 หลายเดือนก่อน +43

    how would a student know when to stop? The final result looks like an intermediate result as well.

    • @declandougan7243
      @declandougan7243 2 หลายเดือนก่อน +10

      Intuition and experience. This isn't an entrance exam or something where enough study is supposed to allow anyone with sufficient time to ace it. It's the Putnam exam, an extremely prestigious and entirely optional endeavor.

    • @LouisLeCrack
      @LouisLeCrack 2 หลายเดือนก่อน +2

      It’s relatively evident that you can’t simplify the sum. If not you probably don’t have the level to pass this exam

    • @aisolutionsindia7138
      @aisolutionsindia7138 2 หลายเดือนก่อน +1

      @@LouisLeCrack are you sure a more elegant expression doesnt exist "at your level"

    • @winstongludovatz111
      @winstongludovatz111 2 หลายเดือนก่อน

      The series converges very fast, therefore numerically useful.

  • @willnewton2247
    @willnewton2247 2 หลายเดือนก่อน +10

    awesome vid! now i just have to go learn about the gamma function lol

  • @improbabilty
    @improbabilty 2 หลายเดือนก่อน +1

    im new but i think your voice is really great!!!!
    no music is no problem!!

    • @Jagoalexander
      @Jagoalexander  2 หลายเดือนก่อน +1

      Thank you, how nice :)

    • @improbabilty
      @improbabilty 2 หลายเดือนก่อน

      @@Jagoalexander you're welcome 🤗
      you got a new subscriber and few views

  • @bigchungusdriplord2301
    @bigchungusdriplord2301 2 หลายเดือนก่อน +5

    You're the perfection of passive agressive lmao

    • @BridgeBum
      @BridgeBum 2 หลายเดือนก่อน

      I was thinking how the contrast between the thanks for all the constructive comments and the selected comment cussing him out was a textbook example of British dry humo(u)r. Chef's kiss.

  • @radiomanroud1559
    @radiomanroud1559 2 หลายเดือนก่อน +1

    Absolutely brillant ! Thank you a lot

    • @Jagoalexander
      @Jagoalexander  2 หลายเดือนก่อน

      Glad you liked it!

  • @Khashayarissi-ob4yj
    @Khashayarissi-ob4yj 2 หลายเดือนก่อน +3

    With luck and more power to you.
    hoping for more videos.

  • @well_said7846
    @well_said7846 2 หลายเดือนก่อน

    The final result is mind blowing great work.😊

  • @paultaufalele1821
    @paultaufalele1821 2 หลายเดือนก่อน

    Woa very cool result. Awesome vid

  • @faramarzsarmadi2398
    @faramarzsarmadi2398 2 หลายเดือนก่อน +1

    Given the behavior at x = 0, we can't integrate from exactly 0. We need to use a limit:
    lim(ε→0+) ∫(ε to ∞) x^(-x) dx

  • @rob876
    @rob876 2 หลายเดือนก่อน

    I liked the music in your last video. I didn't think it was too loud. I thought it was quite calming.
    I, however, did not like your choice of indexing variable in your final answer in this video. Why not just pick n?
    Great video - very clearly explained. Keep up the good work.

  • @nirjharchaudhuri6484
    @nirjharchaudhuri6484 2 หลายเดือนก่อน +4

    I thought the music was incredibly beautiful

  • @alexbenanti5125
    @alexbenanti5125 2 หลายเดือนก่อน +1

    I COULD BARELY HEAR THE MUSIC OVER YOU SOLVING THAT INTEGRAL. SO OBNOXIOUS!!!

    • @Jagoalexander
      @Jagoalexander  2 หลายเดือนก่อน

      I know I'll be quieter next time

  • @faramarzsarmadi2398
    @faramarzsarmadi2398 2 หลายเดือนก่อน +1

    At x = 0, we have 0^0, which is undefined.

  • @조현욱-e5j
    @조현욱-e5j 2 หลายเดือนก่อน +3

    An awsome video it is! I wish your k and u (and possibly n) would be more distinctive.

  • @jessewolf7649
    @jessewolf7649 2 หลายเดือนก่อน +8

    You need a justification for interchanging the infinite sum and the integral at 6:50.

    • @attica7980
      @attica7980 2 หลายเดือนก่อน +2

      The series is uniformly convergent on [0,1], so the interchange of the sum and integral is OK.

    • @bantix9902
      @bantix9902 2 หลายเดือนก่อน +2

      ​@@attica7980 gotta prove that as well 😂

    • @attica7980
      @attica7980 2 หลายเดือนก่อน +1

      @@bantix9902 x ln x has absolute value less than 1 in [0,1], and the sum of 1/n! is convergent; hence the function series is dominated in absolute value by a convergent numerical series, so it is uniformly convergent.

  • @Dissimulate
    @Dissimulate 2 หลายเดือนก่อน

    There is a shorter path between these forms because both are equal to the limit as m goes to infinity of (sum k=1 to m ( ((k/m)^(-k/m))/m ). The 1/m represents the width of each segment of a partial sum in the integral.

  • @GrandAdmiralMitthrawnuruodo
    @GrandAdmiralMitthrawnuruodo 2 หลายเดือนก่อน

    Absolutely incredible. I love those Integrals. Thanks! I‘m surprised that I could solve this with my High school knowledge. :D

  • @zakimoutahir1055
    @zakimoutahir1055 2 หลายเดือนก่อน +4

    Amazing video thanks a lot. I don't know if you know it but there is another integral that's look basically the same called "bernoulli integral", with the only deffencies being the negativ sign in the power. (No worries about the music we all make mistake 🫡🎉)

  • @lucascardoso5332
    @lucascardoso5332 2 หลายเดือนก่อน

    That's funny. I solved this problem while in my second year of undergrad. I was at my friend's place and we had ordered pizza. He had just finished taking his first course on Calculus and I joked that he couldn't find the derivative of x^x, which he did. Then he challenged me to find the integral, which I did too. It was fun!

  • @alwaysfourfun1671
    @alwaysfourfun1671 2 หลายเดือนก่อน

    What does the final summation converge to, e.g. according to Wolfram? Very nice video.

  • @peterwang6474
    @peterwang6474 2 หลายเดือนก่อน

    Ingenious! I wonder if the serial is convergent and if yes, what's the final result?

  • @attica7980
    @attica7980 2 หลายเดือนก่อน +3

    I knew the final equation in the video about the integral being equal to the sum. But how does that answer the question as to what the integral is. It would be good to know how the question was asked, since as stated (find the integral), one cannot know whether the final equation indeed solves the problem.

    • @krishpandey854
      @krishpandey854 2 หลายเดือนก่อน

      I have encountered this question before in a test in India. In that question, they asked the integer value less than or equal to the integral of x^x. So, you had to derive this result (we used reduction because gamma function isn't taught at high school level) and then calculate the first 4 terms to get the approximate value as 129 or something like that

  • @dieguitomvp8338
    @dieguitomvp8338 2 หลายเดือนก่อน

    Easy, full series and substitution

  • @CPILLOW
    @CPILLOW 2 หลายเดือนก่อน +4

    great vid!!!

  • @B00MBASTIC
    @B00MBASTIC 2 หลายเดือนก่อน +1

    Where’s the Family Guy and the Soap Cutting videos on the sides?
    Please consider adding them next video!

    • @Jagoalexander
      @Jagoalexander  2 หลายเดือนก่อน

      I hope this is a joke 😂

  • @VarunDoesMaths
    @VarunDoesMaths 2 หลายเดือนก่อน

    Nice! What mic and software do you use btw?

    • @Jagoalexander
      @Jagoalexander  2 หลายเดือนก่อน

      Goodnotes on iPad and my airpods

  • @looney1023
    @looney1023 2 หลายเดือนก่อน +1

    Is this actually a Putnam question? What's the problem statement? It's a very cool result but how would a Putnam test taker know that the answer is simplified enough at the end, since it can't be evaluated exactly?

    • @Jagoalexander
      @Jagoalexander  2 หลายเดือนก่อน

      The question was show the integral can be written as the sum

  • @hamoudiayoub9341
    @hamoudiayoub9341 2 หลายเดือนก่อน

    Integral is a continuous sum , we solve it by finding the value of that sum but you transfer it to a discreet sum

  • @shreebhattacharjee3502
    @shreebhattacharjee3502 2 หลายเดือนก่อน +3

    amazing!!

  • @mauriziofenderico8348
    @mauriziofenderico8348 2 หลายเดือนก่อน

    Thank you for the excellent demonstration and...NO MUSIC is perfect so one can really concentrate🙂😂

  • @vikivanov5612
    @vikivanov5612 2 หลายเดือนก่อน

    Nice final formula

  • @IcyGM
    @IcyGM 2 หลายเดือนก่อน

    I loved the video and everything, but there's one thing that I would like clarification about. Isn't there any condition on the function that needs to be met in order to interchange the integral and the summation? And if so, was it satisfied here?
    Thank yih

    • @Jagoalexander
      @Jagoalexander  2 หลายเดือนก่อน

      Check my latest video, where I go into detail about this
      I Solved the Impossible Bernoulli Integral!
      th-cam.com/video/T-1fFR8Nk6A/w-d-xo.html

  • @jayxone
    @jayxone 2 หลายเดือนก่อน

    Bro had me so confused with the u’s and k’s looking the exact same

  • @wolfgangroth6265
    @wolfgangroth6265 2 หลายเดือนก่อน

    Thank you for the video. Question for all: Is there any way (similar or not) to calculate the same Integral from 1 to infinity? It must be konvergent, because for X>2 the function x^-x < x^-2 so the Integral is smaller than the Integral of x^-2 wich is konvergent but how to calculate? I’m looking for this for decades…

  • @jimcoughlin4057
    @jimcoughlin4057 2 หลายเดือนก่อน

    I believe you must justify pulling the summation out of the integral. It is not enough to simply say that the sum (-1)^k/n! is a constant; k still appears in the integrand and these 2 operations do not always commute.

  • @skilz8098
    @skilz8098 2 หลายเดือนก่อน

    Perhaps rewrite this integral as being the integration from 0 to 1: x^(i^2 * x) dx
    Maybe this might help. We can think of the exponent: i^2 * x as being periodic where x is the multiple or enumerated counts of a rotation by 180 degrees or PI radians.
    If we look at i^2 as being sqrt(-1) * sqrt(-1) = -1. We know that the value of -1 is a 180-degree rotation of 1. We can use this along with the trigonometric properties to help solve this.
    From what I can see, there is no one exact result from this definite integral. In other words, it's not going to give you a discrete area, volume or region under a curve or manifold. It appears that it is going to give you a series or sequence of them with a specific periodicity determined by the magnitude of x within the exponential component.

  • @jakeaustria5445
    @jakeaustria5445 2 หลายเดือนก่อน +2

    Thank you

  • @alexwarner3803
    @alexwarner3803 20 วันที่ผ่านมา

    What is the source for this solution?
    You and someone else have the exact one, identical to the line, so I was curious if it is in a textbook somewhere.

  • @VeteranVandal
    @VeteranVandal 2 หลายเดือนก่อน +1

    That guy doesn't know the internet is for free.

  • @bryanfranco3943
    @bryanfranco3943 2 หลายเดือนก่อน

    Wouldn't your result mean that the area from 0 to 1 and from 1 to infinity are the same since alpha and x can be taken to be anything, even eachother?? Or am I wrong because you've introduced several changes of variables that places constraints on alpha?

  • @TheFinav
    @TheFinav 2 หลายเดือนก่อน

    Very nice presentation. Thanks.

    • @Jagoalexander
      @Jagoalexander  2 หลายเดือนก่อน

      Glad you liked it!

  • @marcnordin9240
    @marcnordin9240 2 หลายเดือนก่อน +1

    Great video
    Very inspiring

  • @brickie9816
    @brickie9816 2 หลายเดือนก่อน +2

    this video on the other hand had no music. how outrageous. i can't believe you left out such a crucial element of a math video. because of lack of background music, i don't even know if i can call the video informative. i hope you will rethink your life choices that led you to choose to not put music over your video. literally unwatchable
    edit: i skipped the intro and missed you saying there will be no music 😭 not sure if that makes things

  • @winstonridgewayhardy
    @winstonridgewayhardy 2 หลายเดือนก่อน

    I am happy there is no music (but no need to be so angry imho). Wonderfully explained but just a gentle constructive comment is maybe try to write a little more clearly. I know what is going on if I listen along but if I am just looking at the integral I start thinking what is planck's constant popping up in inappropriate place. Haha - just kidding - but you wrote the k really well once so I know you can do it :)

    • @Jagoalexander
      @Jagoalexander  2 หลายเดือนก่อน

      Yes this is a common theme, I'll wrote more clearly from now on 😅

  • @BramCohen
    @BramCohen 2 หลายเดือนก่อน

    There's a strange vagueness to the question here. How do you know that that answer is the final answer and can't be 'simplified' further? What functions would be allowed in that 'simplification'?

  • @Vivir_Astucia
    @Vivir_Astucia 2 หลายเดือนก่อน +2

    I didn't knew we could take out summation out of integrals.....are there conditions for this?

    • @edreeves6440
      @edreeves6440 2 หลายเดือนก่อน

      Absolutely, there are conditions, but if it all works out in the end those conditions probably held.

  • @on_God_
    @on_God_ 2 หลายเดือนก่อน

    I actually liked the music, it jus doesn't have to be loud

  • @thomaslangbein297
    @thomaslangbein297 2 หลายเดือนก่อน +7

    Don’t complain, don’t explain. Just keep to the topic.

  • @imarealperson-5412
    @imarealperson-5412 2 หลายเดือนก่อน

    Good video, but it could use some music. Your soothing voice would work greatly with some jazz or something. Keep up the good content!

  • @sivaramakrishnanm9816
    @sivaramakrishnanm9816 2 หลายเดือนก่อน

    😂well done

  • @ghostsofbeauty.9346
    @ghostsofbeauty.9346 2 หลายเดือนก่อน +2

    3..

    • @Jagoalexander
      @Jagoalexander  2 หลายเดือนก่อน

      3?

    • @Fysiker
      @Fysiker 2 หลายเดือนก่อน

      ​@@Jagoalexander No, 3..

  • @Bodyknock
    @Bodyknock 2 หลายเดือนก่อน

    What was the actual question in the exam? Maybe I missed it but I don’t see the full question in the video or a link to it in the description. Was the question “Show (this integral) equals (this sum)”?

    • @Jagoalexander
      @Jagoalexander  2 หลายเดือนก่อน +2

      Yes it was, the actual question was show it's equal to the sum. Sorry for not explaining that well in the video. I was more focused on the method of solving the integral, the sum converges very quickly to around 1.2..

  • @SeekingTheLoveThatGodMeans7648
    @SeekingTheLoveThatGodMeans7648 2 หลายเดือนก่อน

    When I actually went to school, I was quite bothered by things that were distracting or incomplete, for a grade was going to come to hold me responsible for it just the same. Now, nobody is grading me on following it. Still, some others may be trying to keep up with school, and bringing their concerns with them, and perhaps a MITE irritated with the whole thing, because a grade is going to result at the end. I will not cuss you out, but I will urge you to maximize the helpfulness. k is k. It is not h, it is not u. And if on top of it, some random music is coming, this makes it harder. (Turning the sound off is not a good option, for you are speaking.) Thank you very much for understanding. It's not all about you either.

  • @moonwatcher2001
    @moonwatcher2001 2 หลายเดือนก่อน

    ❤ awesome

  • @מיכאלקונטרוביץ
    @מיכאלקונטרוביץ 2 หลายเดือนก่อน

    Actually you didn't explain why you could interchange the integral and the sum. This is due to the dominated convergence theorem. We must explain all the details why we can do it

  • @DeadJDona
    @DeadJDona 2 หลายเดือนก่อน +2

    1:23 nice voic3 btw

  • @sachin251998
    @sachin251998 2 หลายเดือนก่อน +4

    the way you write k can be interpreted as k, u or even h lol

    • @Jagoalexander
      @Jagoalexander  2 หลายเดือนก่อน +1

      I know, just my handwriting, I'll write more clearly next time!

    • @redgrengrumbholdt2671
      @redgrengrumbholdt2671 2 หลายเดือนก่อน

      my "n" looks exactly like his k

  • @florinleonardpetrescu4677
    @florinleonardpetrescu4677 2 หลายเดือนก่อน +1

    what is sum(0, inf)pow(alfa, -alfa) ? you're just gonna live it like that?

    • @BridgeBum
      @BridgeBum 2 หลายเดือนก่อน

      I suspect there is no closed form answer, but since we are providing feedback: a screenshot to wolframalpha, desmos or the like showing a numerical approximation is a great way to end these type problems.

  • @edreeves6440
    @edreeves6440 2 หลายเดือนก่อน

    Really interesting result but a laboured path through a lot of obvious steps.

    • @Jagoalexander
      @Jagoalexander  2 หลายเดือนก่อน

      Which steps are obvious - I wanted to provide a digestible solution to the average viewer who may not have advanced maths classes in their repertoire

  • @hsfjw
    @hsfjw 2 หลายเดือนก่อน +1

    Use Lambert W function maybe faster?🤔

  • @ezu8501
    @ezu8501 2 หลายเดือนก่อน

    very cool

  • @tfg601
    @tfg601 2 หลายเดือนก่อน

    I find it funny how the first step into finding the derivative and integral of x^-x are the same

  • @marcnordin9240
    @marcnordin9240 2 หลายเดือนก่อน

    Which software do you use for writing down your formulae?

    • @Jagoalexander
      @Jagoalexander  2 หลายเดือนก่อน

      Goodnotes on iPad

  • @hackergaming6869
    @hackergaming6869 2 หลายเดือนก่อน +2

    wouldnt you have to resubstitute in whatever alpha is to get the real result?

    • @joeythreeclubs
      @joeythreeclubs 2 หลายเดือนก่อน +1

      No, alpha is explicitly defined by the infinite sum's bounds

  • @eluoh
    @eluoh 2 หลายเดือนก่อน +1

    no music is fine thanks you !

  • @larzcaetano
    @larzcaetano 2 หลายเดือนก่อน +1

    What kind of sorcery is this

  • @Zaraa-z2e
    @Zaraa-z2e 2 หลายเดือนก่อน +1

    Praise be to God, mathematics is one of my best subjects. I got a score of 97 in the final exam, and the questions were very difficult. I am proud of my score because my students were crying because of the difficulty of the questions. I was hoping to get a score of 100, but I thank God for what I got. ❤

    • @baselinesweb
      @baselinesweb 2 หลายเดือนก่อน

      And we need to know this why? Is your ego sated now?

  • @arekkrolak6320
    @arekkrolak6320 2 หลายเดือนก่อน

    Putting music is fine as long as you are a musician and not talk over it :)

  • @blablub1375
    @blablub1375 2 หลายเดือนก่อน +9

    Changing the order of summation and integration can't be done carelessly, there should have been made some rigorous argument why this holds here.

    • @Jagoalexander
      @Jagoalexander  2 หลายเดือนก่อน +19

      It's called "proof by knowing that it can be done"

    • @xenmaifirebringer552
      @xenmaifirebringer552 2 หลายเดือนก่อน +3

      ​@@Jagoalexander ah, my favourite kind of proof

    • @DeadJDona
      @DeadJDona 2 หลายเดือนก่อน +2

      @@xenmaifirebringer552 it's like breaking a 4th wall, third time is a charm )

    • @LouisLeCrack
      @LouisLeCrack 2 หลายเดือนก่อน +1

      Yup that’s the only hard part, the rest is really easy

    • @LouisLeCrack
      @LouisLeCrack 2 หลายเดือนก่อน

      @@Jagoalexanderyou’re bad bro 😂

  • @Nipppppppppp
    @Nipppppppppp 2 หลายเดือนก่อน

    Nice analysis of the comment lmao
    At any rate, your voice is already music to my ears

  • @krishnamaity5056
    @krishnamaity5056 2 หลายเดือนก่อน

    What answer did official Putnam authority give. Was it this one?

  • @uvogin3143
    @uvogin3143 2 หลายเดือนก่อน +1

    All of this to write 2 times the same stuff

  • @idkusername2981
    @idkusername2981 2 หลายเดือนก่อน

    Hey, I see many comments about the handwriting, but it's completely understandable if you follow along, (I have terrible handwriting too).

  • @Chiavaccio
    @Chiavaccio 2 หลายเดือนก่อน

    👏👏😱👍

  • @potentialofprotonis3
    @potentialofprotonis3 2 หลายเดือนก่อน

    oh my god, fuck that hater
    keep up the good work man

  • @DS-wm6rn
    @DS-wm6rn 2 หลายเดือนก่อน

    I would say the problem solved in the video looks harder than the question asked in the actual exam.
    Namely, asking: "find this integral " does not give you any clues. On the other hand, asking: "prove that this integral equals to this sum" is quite a clue. So the video is a bit misleading/clickbaiting.

  • @yaozhang1687
    @yaozhang1687 2 หลายเดือนก่อน

    Can you solve the integral from 1 to infinity of x^-x?

    • @numbers93
      @numbers93 2 หลายเดือนก่อน

      probably not. You can't swap the infinite sum for the integral if the upper limit of integration is at infinity

    • @yaozhang1687
      @yaozhang1687 2 หลายเดือนก่อน

      @@numbers93 Of cause, you can't solve this integral by infinite sum. But this integral definitely converge.(Because 0 < x^-x < e^-x when x > e, and the integral from e to infinity of e^-x converge) But I can't find any method to calculate the exact value of this integral.

  • @katiekawaii
    @katiekawaii 2 หลายเดือนก่อน

    That's one of the more bizarre comments I've ever seen.

    • @Jagoalexander
      @Jagoalexander  2 หลายเดือนก่อน

      It gave us a good laugh at least

  • @aspirinforbacteria11
    @aspirinforbacteria11 2 หลายเดือนก่อน

    Somehow youtube recommended this video to me and out of curiosity I opened it. Then I realized that this is the same person whom I learned Feynman Technique from lol! Subscribed!!

    • @Jagoalexander
      @Jagoalexander  2 หลายเดือนก่อน

      Thanks mate 👍 👌

  • @ethanbartiromo2888
    @ethanbartiromo2888 2 หลายเดือนก่อน +1

    So, a question I have is was the question on the Putnam in the form of “prove that \int_0^1 x^{-x}= \sum_{k = 1}^{\infty} k^{-k}”? Otherwise, I don’t think this would be a valid solution to the problem because it still isn’t in a constant form, it’s in the form of an infinite sum, which still needs different methods to solve/approximate. I took the Putnam twice, once in my freshman year of college, and again in my junior year, and unless it was worded like that, this would probably get a 1 or 4 or something like that, not a full 10

    • @Jagoalexander
      @Jagoalexander  2 หลายเดือนก่อน +2

      Yes, the question was show that the integral can be written as the infinite sum. The infinite sum converges very quickly to around 1.2… not an exact value unfortunately

  • @DeadJDona
    @DeadJDona 2 หลายเดือนก่อน +1

    9:00 why'd u keep saying k when writing n? or write n each time you say k?
    geez writing is difficult (c) 3b1b

    • @DeadJDona
      @DeadJDona 2 หลายเดือนก่อน

      13:00 k

    • @Jagoalexander
      @Jagoalexander  2 หลายเดือนก่อน +3

      Both n and k are common variables to use with sums, however I've opted to use k as my variable I'm this video. In the video I do write the letter k but my handwriting makes it look like an n, making it confusing. But to clarify, I'm both saying and writing k

    • @DeadJDona
      @DeadJDona 2 หลายเดือนก่อน

      @@Jagoalexander it would be easier i guess to just say n ))

    • @_Heb_
      @_Heb_ 2 หลายเดือนก่อน +1

      @@DeadJDonaBut then you'd be writing k and saying n!

    • @DeadJDona
      @DeadJDona 2 หลายเดือนก่อน

      @@_Heb_ how would you know that he's writing k then? )) or k factorial?

  • @fderty4
    @fderty4 2 หลายเดือนก่อน

    I could solve this integral in the bat of an eye. I chose not to 😎

  • @nealkuril3588
    @nealkuril3588 2 หลายเดือนก่อน

    4:50

  • @MrNibiru2112
    @MrNibiru2112 2 หลายเดือนก่อน

    Im afraid, Factorization of the constant at 6:26 is not correct